Solving integral - gaussian distribution of cos

Click For Summary
The discussion centers on proving an integral involving a cosine function and a Gaussian distribution. The integral can be simplified by changing the limits from negative to positive infinity, given that sigma is much smaller than L_av. Participants suggest looking up known integral solutions and consider using complex exponentials to simplify the cosine term. Clarifications are requested regarding the notation used for L_av and the structure of the expressions to avoid confusion. The goal remains to establish the proportional relationship between the integral and the specified cosine and exponential terms.
poul
Messages
15
Reaction score
0

Homework Statement



I have to prove:

∫(-infinity:infinity) cos(pi*v/2L)*e^-((L-L_av)^2/sqrt(2pi)*sigma^2) dL proportional to
cos(pi*v/2L_av)*e^-(t/tau)^2

tau is some constant, and sigma << L_av.

The Attempt at a Solution



i can change the integral to 0:infinity, since sigma << L_av. Then i have to look up some integral solution, probably:

∫(0:infinity) cos(bx)*e^-ax^2 dx

I assume i have to do some trick like 1/L approximately L/L_av^2 - but how can i justify that?
 
Physics news on Phys.org
Can you use complex numbers? If so, you can rewrite your cosine in term of complex exponentials which will make things quite easy.
 
poul said:

Homework Statement



I have to prove:

∫(-infinity:infinity) cos(pi*v/2L)*e^-((L-L_av)^2/sqrt(2pi)*sigma^2) dL proportional to
cos(pi*v/2L_av)*e^-(t/tau)^2

tau is some constant, and sigma << L_av.

The Attempt at a Solution



i can change the integral to 0:infinity, since sigma << L_av. Then i have to look up some integral solution, probably:

∫(0:infinity) cos(bx)*e^-ax^2 dx

I assume i have to do some trick like 1/L approximately L/L_av^2 - but how can i justify that?

It is very unclear what your original problem is, because you do not use brackets. You have something written as L_av. Does this mean Lav or does it mean Lav? If you mean the first, write L_{av} or L_(av), and if you mean the second, either write a*L_v or (L_v)a. Better still, use the "X2" button in the menu above the input panel. Anyway, setting L_av = c, some constant, it is still not clear whether you mean
\cos \left( \frac{\pi v}{2L} \right) \text{ or } \cos \left( \frac{\pi v}{2} L \right) in the integrand. If you mean the first, write cos(pi*v/(2L)), but if you mean the latter, write cos((pi*v/2)L).

Similarly, when you write e^-ax^2, you are literally writing ##e^{-a} x^2## if we read it using standard rules and conventions. I guess you mean ##e^{-ax^2},## which is e^{-ax^2} or e^(-ax^2) in plain text, or better still, e-ax^2 or e-ax2, using the "X2" button in the menu at the top of the input panel.

RGV
 
Okay, I have to prove:

∫(-infinity:infinity) cos(pi*v/(2L))*e-((L-L_{av})^2/(2*sigma^2)) dL proportional to
cos(pi*v/(2L_{av}))*e-(t/tau)^2

sigma << L_{av} - both positive
 
Question: A clock's minute hand has length 4 and its hour hand has length 3. What is the distance between the tips at the moment when it is increasing most rapidly?(Putnam Exam Question) Answer: Making assumption that both the hands moves at constant angular velocities, the answer is ## \sqrt{7} .## But don't you think this assumption is somewhat doubtful and wrong?

Similar threads

  • · Replies 9 ·
Replies
9
Views
3K
  • · Replies 4 ·
Replies
4
Views
1K
  • · Replies 6 ·
Replies
6
Views
2K
  • · Replies 1 ·
Replies
1
Views
2K
  • · Replies 4 ·
Replies
4
Views
2K
  • · Replies 10 ·
Replies
10
Views
2K
  • · Replies 5 ·
Replies
5
Views
2K
  • · Replies 3 ·
Replies
3
Views
2K
  • · Replies 2 ·
Replies
2
Views
1K
  • · Replies 5 ·
Replies
5
Views
2K